Question and Answers Forum

All Questions      Topic List

Number Theory Questions

Previous in All Question      Next in All Question      

Previous in Number Theory      Next in Number Theory      

Question Number 108516 by Rasheed.Sindhi last updated on 17/Aug/20

If x+(1/x)=2(x≠0), prove that       x^n +(1/x^n )=2  ∀ n∈ Z

$$\mathrm{If}\:\mathrm{x}+\frac{\mathrm{1}}{\mathrm{x}}=\mathrm{2}\left(\mathrm{x}\neq\mathrm{0}\right),\:\mathrm{prove}\:\mathrm{that} \\ $$$$\:\:\:\:\:\mathrm{x}^{\mathrm{n}} +\frac{\mathrm{1}}{\mathrm{x}^{\mathrm{n}} }=\mathrm{2}\:\:\forall\:{n}\in\:\mathbb{Z} \\ $$

Commented by mr W last updated on 17/Aug/20

if x>0:  x+(1/x)≥2(√(x×(1/x)))=2  = only for x=(1/x)=1    if x<0:  x+(1/x)=−(−x+(1/(−x)))≤−2  ⇒no solution for x+(1/x)=2    ⇒only solution for x+(1/x)=2 is  x=1    ⇒x^n +(1/x^n )=1+1=2

$${if}\:{x}>\mathrm{0}: \\ $$$${x}+\frac{\mathrm{1}}{{x}}\geqslant\mathrm{2}\sqrt{{x}×\frac{\mathrm{1}}{{x}}}=\mathrm{2} \\ $$$$=\:{only}\:{for}\:{x}=\frac{\mathrm{1}}{{x}}=\mathrm{1} \\ $$$$ \\ $$$${if}\:{x}<\mathrm{0}: \\ $$$${x}+\frac{\mathrm{1}}{{x}}=−\left(−{x}+\frac{\mathrm{1}}{−{x}}\right)\leqslant−\mathrm{2} \\ $$$$\Rightarrow{no}\:{solution}\:{for}\:{x}+\frac{\mathrm{1}}{{x}}=\mathrm{2} \\ $$$$ \\ $$$$\Rightarrow{only}\:{solution}\:{for}\:{x}+\frac{\mathrm{1}}{{x}}=\mathrm{2}\:{is} \\ $$$${x}=\mathrm{1} \\ $$$$ \\ $$$$\Rightarrow{x}^{{n}} +\frac{\mathrm{1}}{{x}^{{n}} }=\mathrm{1}+\mathrm{1}=\mathrm{2} \\ $$

Commented by Rasheed.Sindhi last updated on 17/Aug/20

e^x cellent!

$$\boldsymbol{\mathrm{e}}^{\boldsymbol{\mathrm{x}}} \mathrm{cellent}! \\ $$

Commented by udaythool last updated on 17/Aug/20

By induction it follows...

$$\mathrm{By}\:\mathrm{induction}\:\mathrm{it}\:\mathrm{follows}... \\ $$

Commented by Rasheed.Sindhi last updated on 17/Aug/20

Thanks sir waiting for proof by  induction....

$${Thanks}\:{sir}\:{waiting}\:{for}\:{proof}\:{by} \\ $$$${induction}.... \\ $$

Commented by udaythool last updated on 17/Aug/20

Let n=0,1,...  Induction statment:  P_n : x^n +(1/x^n )=2  For n=0 and 1, P_n  is true  Induction hypothesis:  Let P_n  is true for all n=0,..., k  ∴(x^(k+1) +(1/x^(k+1) ))=(x+(1/x))(x^k +(1/x^k ))−2=2x2−2=2  i.e. P_k ⇒P_(k+1)   Thus by PI P_n  is true for all n∈Z^+   If n∈Z^− ⇒n=−m for m=∣n∣∈Z^+   ∴ x^n +(1/x^n )=x^(−m) +(1/x^(−m) )=x^m +(1/x^m )=2  And hence P_n  is true ∀n∈Z  etc...

$$\mathrm{Let}\:{n}=\mathrm{0},\mathrm{1},... \\ $$$$\mathrm{Induction}\:\mathrm{statment}: \\ $$$${P}_{\mathrm{n}} :\:{x}^{{n}} +\frac{\mathrm{1}}{{x}^{{n}} }=\mathrm{2} \\ $$$$\mathrm{For}\:{n}=\mathrm{0}\:\mathrm{and}\:\mathrm{1},\:{P}_{{n}} \:\mathrm{is}\:\mathrm{true} \\ $$$$\mathrm{Induction}\:\mathrm{hypothesis}: \\ $$$$\mathrm{Let}\:{P}_{{n}} \:\mathrm{is}\:\mathrm{true}\:\mathrm{for}\:\mathrm{all}\:{n}=\mathrm{0},...,\:{k} \\ $$$$\therefore\left({x}^{{k}+\mathrm{1}} +\frac{\mathrm{1}}{{x}^{{k}+\mathrm{1}} }\right)=\left({x}+\frac{\mathrm{1}}{{x}}\right)\left({x}^{{k}} +\frac{\mathrm{1}}{{x}^{{k}} }\right)−\mathrm{2}=\mathrm{2x2}−\mathrm{2}=\mathrm{2} \\ $$$$\mathrm{i}.\mathrm{e}.\:{P}_{\mathrm{k}} \Rightarrow{P}_{{k}+\mathrm{1}} \\ $$$$\mathrm{Thus}\:\mathrm{by}\:\mathrm{PI}\:{P}_{{n}} \:\mathrm{is}\:\mathrm{true}\:\mathrm{for}\:\mathrm{all}\:{n}\in\mathbb{Z}^{+} \\ $$$$\mathrm{If}\:{n}\in\mathbb{Z}^{−} \Rightarrow{n}=−{m}\:\mathrm{for}\:{m}=\mid{n}\mid\in\mathbb{Z}^{+} \\ $$$$\therefore\:{x}^{{n}} +\frac{\mathrm{1}}{{x}^{{n}} }={x}^{−{m}} +\frac{\mathrm{1}}{{x}^{−{m}} }={x}^{{m}} +\frac{\mathrm{1}}{{x}^{{m}} }=\mathrm{2} \\ $$$$\mathrm{And}\:\mathrm{hence}\:{P}_{{n}} \:\mathrm{is}\:\mathrm{true}\:\forall{n}\in\mathbb{Z} \\ $$$$\mathrm{etc}... \\ $$$$ \\ $$

Commented by Rasheed.Sindhi last updated on 18/Aug/20

Nice sir!   The line below needs some  detail: Thanks.  ∴(x^(k+1) +(1/x^(k+1) ))=(x+(1/x))(x^k +(1/x^k ))−2=2x2−2=2

$${Nice}\:{sir}!\: \\ $$$${The}\:{line}\:{below}\:{needs}\:{some} \\ $$$${detail}:\:\mathcal{T}{hanks}. \\ $$$$\therefore\left({x}^{{k}+\mathrm{1}} +\frac{\mathrm{1}}{{x}^{{k}+\mathrm{1}} }\right)=\left({x}+\frac{\mathrm{1}}{{x}}\right)\left({x}^{{k}} +\frac{\mathrm{1}}{{x}^{{k}} }\right)−\mathrm{2}=\mathrm{2x2}−\mathrm{2}=\mathrm{2} \\ $$$$ \\ $$

Commented by udaythool last updated on 13/Sep/20

Actually;  (x+(1/x))(x^k +(1/x^k ))=(x^(k+1) +(1/x^(k+1) ))+(x^(k−1) +(1/x^(k−1) ))  ⇒(x^(k+1) +(1/x^(k+1) ))=(x+(1/x))(x^k +(1/x^k ))−(x^(k−1) +(1/x^(k−1) ))  we have assumed (induction  hypothesis) p_n  is true for n=k  i.e. p_n =2 for n=1,2,…,k  Thus (x^(k+1) +(1/x^(k+1) ))=2x2−2=2

$$\mathrm{Actually}; \\ $$$$\left({x}+\frac{\mathrm{1}}{{x}}\right)\left({x}^{{k}} +\frac{\mathrm{1}}{{x}^{{k}} }\right)=\left({x}^{{k}+\mathrm{1}} +\frac{\mathrm{1}}{{x}^{{k}+\mathrm{1}} }\right)+\left({x}^{{k}−\mathrm{1}} +\frac{\mathrm{1}}{{x}^{{k}−\mathrm{1}} }\right) \\ $$$$\Rightarrow\left({x}^{{k}+\mathrm{1}} +\frac{\mathrm{1}}{{x}^{{k}+\mathrm{1}} }\right)=\left({x}+\frac{\mathrm{1}}{{x}}\right)\left({x}^{{k}} +\frac{\mathrm{1}}{{x}^{{k}} }\right)−\left({x}^{{k}−\mathrm{1}} +\frac{\mathrm{1}}{{x}^{{k}−\mathrm{1}} }\right) \\ $$$$\mathrm{we}\:\mathrm{have}\:\mathrm{assumed}\:\left(\mathrm{induction}\right. \\ $$$$\left.\mathrm{hypothesis}\right)\:\mathrm{p}_{\mathrm{n}} \:\mathrm{is}\:\mathrm{true}\:\mathrm{for}\:\mathrm{n}=\mathrm{k} \\ $$$$\mathrm{i}.\mathrm{e}.\:\mathrm{p}_{\mathrm{n}} =\mathrm{2}\:\mathrm{for}\:\mathrm{n}=\mathrm{1},\mathrm{2},\ldots,\mathrm{k} \\ $$$$\mathrm{Thus}\:\left({x}^{{k}+\mathrm{1}} +\frac{\mathrm{1}}{{x}^{{k}+\mathrm{1}} }\right)=\mathrm{2x2}−\mathrm{2}=\mathrm{2} \\ $$

Answered by floor(10²Eta[1]) last updated on 17/Aug/20

x^2 −2x+1=(x−1)^2 =0⇒x=1  ⇒x^n =1∴x^n +(1/x^n )=2=1+(1/1)

$$\mathrm{x}^{\mathrm{2}} −\mathrm{2x}+\mathrm{1}=\left(\mathrm{x}−\mathrm{1}\right)^{\mathrm{2}} =\mathrm{0}\Rightarrow\mathrm{x}=\mathrm{1} \\ $$$$\Rightarrow\mathrm{x}^{\mathrm{n}} =\mathrm{1}\therefore\mathrm{x}^{\mathrm{n}} +\frac{\mathrm{1}}{\mathrm{x}^{\mathrm{n}} }=\mathrm{2}=\mathrm{1}+\frac{\mathrm{1}}{\mathrm{1}} \\ $$

Commented by Rasheed.Sindhi last updated on 17/Aug/20

θαnX sir!

$$\theta\alpha{n}\mathcal{X}\:{sir}! \\ $$

Answered by mathmax by abdo last updated on 17/Aug/20

x+(1/x) =2 ⇒x^2 +1 =2x ⇒x^2 −2x+1 =0 ⇒(x−1)^2  =0 ⇒x=1 ⇒  ∀n    we hsve   x^n  +(1/x^n ) =2

$$\mathrm{x}+\frac{\mathrm{1}}{\mathrm{x}}\:=\mathrm{2}\:\Rightarrow\mathrm{x}^{\mathrm{2}} +\mathrm{1}\:=\mathrm{2x}\:\Rightarrow\mathrm{x}^{\mathrm{2}} −\mathrm{2x}+\mathrm{1}\:=\mathrm{0}\:\Rightarrow\left(\mathrm{x}−\mathrm{1}\right)^{\mathrm{2}} \:=\mathrm{0}\:\Rightarrow\mathrm{x}=\mathrm{1}\:\Rightarrow \\ $$$$\forall\mathrm{n}\:\:\:\:\mathrm{we}\:\mathrm{hsve}\:\:\:\mathrm{x}^{\mathrm{n}} \:+\frac{\mathrm{1}}{\mathrm{x}^{\mathrm{n}} }\:=\mathrm{2} \\ $$

Commented by Rasheed.Sindhi last updated on 18/Aug/20

Thanx sir

$${Thanx}\:{sir} \\ $$

Terms of Service

Privacy Policy

Contact: info@tinkutara.com